Difference between revisions of "2018 AMC 12A Problems/Problem 2"

(Solution 3: Deleted the repetitive solution. For this problem, I think two solutions are sufficient. Many students will take Sol 2.)
(Solution 2)
Line 14: Line 14:
  
 
== Solution 2 ==
 
== Solution 2 ==
The value of <math>5</math>-pound rocks is <math>\$14\div5=\$2.80</math> per pound, and the value of <math>4</math>-pound rocks is <math>\$11\div4=\$2.75</math> per pound. Intuitively, we wish to maximize the number of <math>5</math>-pound rocks and minimize the number of <math>1</math>-pound rocks. We have two cases:
+
<b>NOT DONE YET. WILL FINISH UP. NO EDIT PLEASE.</b>
<ol style="margin-left: 1.5em;">
+
 
  <li>We get three <math>5</math>-pound rocks and three <math>1</math>-pound rocks, for a total value of <math>\$14\cdot3+\$2\cdot3=\$48.</math></li><p>
+
The value of <math>5</math>-pound rocks is <math>\$14\div5=\$2.80</math> per pound, and the value of <math>4</math>-pound rocks is <math>\$11\div4=\$2.75</math> per pound. Clearly, Carl should not carry more than three <math>1</math>-pound rocks. Otherwise, he can replace the <math>1</math>-pound rocks with the heavier rocks, preserving the weight but increasing the total value.  
  <li>We get two <math>5</math>-pound rocks and two <math>4</math>-pound rocks, for a total value of <math>\$14\cdot2+\$11\cdot2=\$50.</math></li><p>
+
 
</ol>
+
We perform casework on the number of <math>1</math>-pound rocks that Carl can carry out:
Clearly, Case 2 produces the maximum total value. So, the answer is <math>\boxed{\textbf{(C) } 50}.</math>
+
<cmath>\begin{array}{c|c|c||c}
 +
& &  \\ [-2.5ex]
 +
\boldsymbol{1}\textbf{-Pound Rocks} & \boldsymbol{4}\textbf{-Pound Rocks} & \boldsymbol{5}\textbf{-Pound Rocks} \\
 +
\textbf{(}\boldsymbol{\$2}\textbf{ Each)} & \textbf{(}\boldsymbol{\$11}\textbf{ Each)} & \textbf{(}\boldsymbol{\$14}\textbf{ Each)} \\ [0.5ex]
 +
\hline
 +
& & \\ [-2ex]
 +
\textbf{Alice} & [6,\infty) & [0,6) \\
 +
& & \\ [-2.25ex]
 +
\textbf{Bob} & [0,5] & (5,\infty)  \\
 +
& & \\ [-2.25ex]
 +
\textbf{Charlie} & [0,4] & (4,\infty)
 +
\end{array}</cmath>
  
 
<u><b>Remark</b></u>
 
<u><b>Remark</b></u>

Revision as of 01:18, 25 August 2021

Problem

While exploring a cave, Carl comes across a collection of $5$-pound rocks worth $$14$ each, $4$-pound rocks worth $$11$ each, and $1$-pound rocks worth $$2$ each. There are at least $20$ of each size. He can carry at most $18$ pounds. What is the maximum value, in dollars, of the rocks he can carry out of the cave?

$\textbf{(A) } 48 \qquad \textbf{(B) } 49 \qquad \textbf{(C) } 50 \qquad \textbf{(D) } 51 \qquad \textbf{(E) } 52$

Solution 1

Since each rock is worth $1$ dollar less than $3$ times its weight (in pounds), the answer is just $3\cdot 18=54$ minus the minimum number of rocks we need to make $18$ pounds. Note that we need at least $4$ rocks (two $5$-pound rocks and two $4$-pound rocks) to make $18$ pounds, so the answer is $54-4=\boxed{\textbf{(C) } 50}.$

~Kevindujin (Solution)

~MRENTHUSIASM (Revision)

Solution 2

NOT DONE YET. WILL FINISH UP. NO EDIT PLEASE.

The value of $5$-pound rocks is $$14\div5=$2.80$ per pound, and the value of $4$-pound rocks is $$11\div4=$2.75$ per pound. Clearly, Carl should not carry more than three $1$-pound rocks. Otherwise, he can replace the $1$-pound rocks with the heavier rocks, preserving the weight but increasing the total value.

We perform casework on the number of $1$-pound rocks that Carl can carry out: \[\begin{array}{c|c|c||c} & &  \\ [-2.5ex] \boldsymbol{1}\textbf{-Pound Rocks} & \boldsymbol{4}\textbf{-Pound Rocks} & \boldsymbol{5}\textbf{-Pound Rocks} \\ \textbf{(}\boldsymbol{$2}\textbf{ Each)} & \textbf{(}\boldsymbol{$11}\textbf{ Each)} & \textbf{(}\boldsymbol{$14}\textbf{ Each)} \\ [0.5ex] \hline & & \\ [-2ex] \textbf{Alice} & [6,\infty) & [0,6) \\  & & \\ [-2.25ex] \textbf{Bob} & [0,5] & (5,\infty)  \\ & & \\ [-2.25ex] \textbf{Charlie} & [0,4] & (4,\infty) \end{array}\]

Remark

Note that an upper bound of the total value is $$2.80\cdot18=$50.40,$ from which we can eliminate choices $\textbf{(D)}$ and $\textbf{(E)}.$

~Pyhm2017 (Fundamental Logic)

~MRENTHUSIASM (Reconstruction)

See Also

2018 AMC 12A (ProblemsAnswer KeyResources)
Preceded by
Problem 1
Followed by
Problem 3
1 2 3 4 5 6 7 8 9 10 11 12 13 14 15 16 17 18 19 20 21 22 23 24 25
All AMC 12 Problems and Solutions

The problems on this page are copyrighted by the Mathematical Association of America's American Mathematics Competitions. AMC logo.png